The products scheduled for the last four sales, in order from the fourth sale to the seventh, could be

AK11 on June 18, 2020

Set up

Can I please get some help with the set up of this game? Thank you.

Replies
Create a free account to read and take part in forum discussions.

Already have an account? log in

Skylar on June 19, 2020

@AK11, happy to help!

HLMPRST
__ __ __ __ __ __ __
1 2 3 4 5 6 7

Rule #1: H __ __ + S or S __ __ + H
Rule #2: PS or SP
Rule #3: T = 1 or 7
Rule #4: not T1 (so T7) -> R1
not R1 -> T1 (so not T7)
Rule #5: L - H

This gives us two basic framework options:
(1) T __ __ __ __ __ __
1 2 3 4 5 6 7
(2) R __ __ __ __ __ T
1 2 3 4 5 6 7

As for this specific question, we can go through rule by rule to eliminate answer choices. Remember that this question refers to spots 4-7.

(B) violates Rule #2 because P is not next to S.
(D) violates Rule #3 because T is 6th.
(E) violates Rule #4 because T is not first and neither is R.
(A) violates Rule #5 because L cannot precede H.
(C) is therefore correct.

Does that make sense? Hope it helps! Please let us know if you have any other questions!

AK11 on June 24, 2020

Thank you, this was very helpful